LSAT and Law School Admissions Forum

Get expert LSAT preparation and law school admissions advice from PowerScore Test Preparation.

User avatar
 Dave Killoran
PowerScore Staff
  • PowerScore Staff
  • Posts: 5852
  • Joined: Mar 25, 2011
|
#41570
Complete Question Explanation
(The complete setup for this game can be found here: lsat/viewtopic.php?t=7106)

The correct answer choice is (B)

This question suspends and then replaces the third rule, forcing you to re-create the initial setup with a new rule. This is the resulting setup:
F92_Game_#3_#17_diagram 1.png
Because the question stem asks for what is not necessarily true, you should immediately look for answers that include 1 and 3 or 2 and 5 because those are the only uncertainties in the diagram above. Answer choice (B) neatly plays on the uncertainty surrounding 2 and 5, and (B) is the correct answer.
You do not have the required permissions to view the files attached to this post.
 hunterama1
  • Posts: 16
  • Joined: Oct 23, 2012
|
#6221
Re: Question 17

8>4>6 84X2/55/2
1/33/16*7X
M T W Th Fr

In the powerscore online test prep book which is the same as the encyclopedia 1 explanation, it merely asserts: "This question suspends and then replaces the third rule, forcing you to re-creae the initial setup with a new rule. This is the resulting setup (above).

So that's not exactly an explanation. The rule of 8>4>6 allows for 6 to also in the slots where 2/5, 5/2 are above. I see no limiting rule disallowing this. It a Not Necessarily True question so one looks to the dual options as those are the only uncertainties however it appears to me that 6 would or should be included in this uncertainty.

Please explain why 6 cannot be placed in Th and Fr
User avatar
 Dave Killoran
PowerScore Staff
  • PowerScore Staff
  • Posts: 5852
  • Joined: Mar 25, 2011
|
#6222
Hi Hunter,

Thanks for the question. In this problem, let's consider what happens when 6 is placed on Thursday and Friday. Since Thursday morning is already 7, and Friday morning is already unavailable, 6 would have to be cleaned in the afternoon, correct? On its face, this seems possible, but consider for a moment how this affects the variables in the last rule that must be cleaned in the afternoon. What's left for them?

Initially we have five available afternoon spaces, and we determined 8 must be cleaned on Monday, in the afternoon from the last rule. That doesn't change here. From the rule change, 4 is now cleaned on Tuesday afternoon. And from the second rule, no street is cleaned on Wednesday afternoon. Thus, only Thursday afternoon and Friday afternoon remain for 2 and 5, creating 2/5 dual-options on both. If you were to place 6 on Thursday or Friday, you would end up violating the final rule by forcing 2 or 5 to be cleaned in the morning of one of the earlier days.

Whenever you see a situation that isn't immediately clear to you (as in a Not Law that doesn't make sense, or a placement of a variable in a certain position), one way to solve those situations is to try to contradict those placements (for Not Laws, place the variable there; for variables placed, try to place them elsewhere). Using that approach often cements the idea more powerfully than any explanation.

Please let me know if that helps. Thanks!
 hunterama1
  • Posts: 16
  • Joined: Oct 23, 2012
|
#6238
Thank you for the prompt response to my quandary. What luck- I went straight to the top on my first post!

I get it.

For some (momentary lapse of) reason I was trying to place three variables (2,5,6) into two afternoon slots negating physics that only one variable can be placed per slot and that 2/5 can not go in the mornings thereby forcing 6 only to Wed morning.

Great prep materials and program.

Thanks again.

Hunter
Targeting 175+ Dec '12 or Feb '13
 afulbright.2000@gmail.com
  • Posts: 13
  • Joined: Apr 28, 2021
|
#97205
I am slightly confused on this question and would love to process this with one of you-

So, given the setup:
8 4 X 2/5 5/2
1/3 3/1 6 7 X

For the same reason that B is true because the crew could clean the Fifth street before the Second street OR Second before Fifth giving either possibility the chance to be false, isn't answer choice E playing to that same logic?

Depending on if the top row reads: 8 4 X 2 5 or 8 4 X 5 2, one of the possibilities would allow for the seventh street to be before the Second street and the other would not (because they would be on the same day)... So, why doesn't E qualify as an answer that could be false?
 Adam Tyson
PowerScore Staff
  • PowerScore Staff
  • Posts: 5153
  • Joined: Apr 14, 2011
|
#97258
Same day, yes, but 7th would be in the morning of that day and 2nd would be in the afternoon, and morning is still before afternoon of the same day. The sequencing is not just from one day to the next, but also within each day, with Monday morning being first, Monday afternoon being second, Tuesday morning being third, etc.

Get the most out of your LSAT Prep Plus subscription.

Analyze and track your performance with our Testing and Analytics Package.